Real integral using residue theorem - why doesn't this work? The Next CEO of Stack OverflowMistake with using residue theory for calculating $int_-infty^inftyfracsin(x)xdx$Evaluating Integral with Residue TheoremReal Pole Residue theoremSolving this complicated integral using the Residue TheoremIntegrating secans over the imaginary axis using the residue theoremWhy doesn't this residue method work for calculating $sum_k=1^k=infty fraccos(k x)k^2$Compute integral using residue theoremEvaluating a real definite integral using residue theoremCalculating this integral using Residue TheoremCalculating integrals using the residue theoremsolving integral with real exponent and real pole with residue theorem

Do I need to enable Dev Hub in my PROD Org?

What is the result of assigning to std::vector<T>::begin()?

MessageLevel in QGIS3

What happened in Rome, when the western empire "fell"?

Preparing Indesign booklet with .psd graphics for print

Make solar eclipses exceedingly rare, but still have new moons

To not tell, not take, and not want

Should I tutor a student who I know has cheated on their homework?

If a black hole is created from light, can this black hole then move at speed of light?

Bold, vivid family

Why has the US not been more assertive in confronting Russia in recent years?

Can you replace a racial trait cantrip when leveling up?

Is it ever safe to open a suspicious html file (e.g. email attachment)?

What's the best way to handle refactoring a big file?

What is "(CFMCC)" on an ILS approach chart?

Is there a way to save my career from absolute disaster?

In excess I'm lethal

How do scammers retract money, while you can’t?

How do I transpose the 1st and -1th levels of an arbitrarily nested array?

Does it take more energy to get to Venus or to Mars?

Inappropriate reference requests from Journal reviewers

Return the Closest Prime Number

Received an invoice from my ex-employer billing me for training; how to handle?

Unreliable Magic - Is it worth it?



Real integral using residue theorem - why doesn't this work?



The Next CEO of Stack OverflowMistake with using residue theory for calculating $int_-infty^inftyfracsin(x)xdx$Evaluating Integral with Residue TheoremReal Pole Residue theoremSolving this complicated integral using the Residue TheoremIntegrating secans over the imaginary axis using the residue theoremWhy doesn't this residue method work for calculating $sum_k=1^k=infty fraccos(k x)k^2$Compute integral using residue theoremEvaluating a real definite integral using residue theoremCalculating this integral using Residue TheoremCalculating integrals using the residue theoremsolving integral with real exponent and real pole with residue theorem










2












$begingroup$


Consider the following definite real integral:
$$I = int_0^infty dx frace^-ix - e^ixx$$



Using the $textSi(x)$ function, I can solve it easily,
$$I = -2i int_0^infty dx frace^-ix - e^ix-2ix = -2i int_0^infty dx fracsinxx = -2i lim_x to infty textSi(x) = -2i left(fracpi2right) = - i pi,$$
simply because I happen to know that $mathrmSi(x)$ asymptotically approaches $pi/2$.



However, if I try to calculate it using the residue theorem, I get the wrong answer, off by a factor of $2$ and I'm not sure if I understand why. Here's the procedure:
$$I= int_0^infty dx frace^-ixx - int_0^infty dx frac e^ixx = colorred-int_-infty^0 dx frace^ixx - int_0^infty dx frac e^ixx
= -int_-infty^infty dx frace^ixx $$

Then I define $$I_epsilon := -int_-infty^infty dx frace^ixx-ivarepsilon$$ for $varepsilon > 0$ so that$$I=lim_varepsilon to 0^+ I_varepsilon.$$
Then I complexify the integration variable and integrate over a D-shaped contour over the upper half of the complex plane. I choose that contour because
$$lim_x to +iinfty frace^ixx-ivarepsilon = 0$$ and it contains the simple pole at $x_0 = i varepsilon$. Using the residue theorem with the contour enclosing $x_0$ $$I_varepsilon = -2 pi i , textRes_x_0 left( frace^ixx-ivarepsilonright) = -2 pi i left( frace^ix1 right)Biggrvert_x=x_0=ivarepsilon=-2 pi i , e^-varepsilon.$$
Therefore,
$$I=lim_varepsilon to 0^+ left( -2 pi i , e^-varepsilon right) = -2pi i.$$



However, that is obviously wrong. Where exactly is the mistake?










share|cite|improve this question









$endgroup$







  • 1




    $begingroup$
    math.stackexchange.com/a/2270510/155436
    $endgroup$
    – Count Iblis
    4 hours ago










  • $begingroup$
    @CountIblis Didn't catch that one before, thank you!
    $endgroup$
    – Ivan V.
    4 hours ago















2












$begingroup$


Consider the following definite real integral:
$$I = int_0^infty dx frace^-ix - e^ixx$$



Using the $textSi(x)$ function, I can solve it easily,
$$I = -2i int_0^infty dx frace^-ix - e^ix-2ix = -2i int_0^infty dx fracsinxx = -2i lim_x to infty textSi(x) = -2i left(fracpi2right) = - i pi,$$
simply because I happen to know that $mathrmSi(x)$ asymptotically approaches $pi/2$.



However, if I try to calculate it using the residue theorem, I get the wrong answer, off by a factor of $2$ and I'm not sure if I understand why. Here's the procedure:
$$I= int_0^infty dx frace^-ixx - int_0^infty dx frac e^ixx = colorred-int_-infty^0 dx frace^ixx - int_0^infty dx frac e^ixx
= -int_-infty^infty dx frace^ixx $$

Then I define $$I_epsilon := -int_-infty^infty dx frace^ixx-ivarepsilon$$ for $varepsilon > 0$ so that$$I=lim_varepsilon to 0^+ I_varepsilon.$$
Then I complexify the integration variable and integrate over a D-shaped contour over the upper half of the complex plane. I choose that contour because
$$lim_x to +iinfty frace^ixx-ivarepsilon = 0$$ and it contains the simple pole at $x_0 = i varepsilon$. Using the residue theorem with the contour enclosing $x_0$ $$I_varepsilon = -2 pi i , textRes_x_0 left( frace^ixx-ivarepsilonright) = -2 pi i left( frace^ix1 right)Biggrvert_x=x_0=ivarepsilon=-2 pi i , e^-varepsilon.$$
Therefore,
$$I=lim_varepsilon to 0^+ left( -2 pi i , e^-varepsilon right) = -2pi i.$$



However, that is obviously wrong. Where exactly is the mistake?










share|cite|improve this question









$endgroup$







  • 1




    $begingroup$
    math.stackexchange.com/a/2270510/155436
    $endgroup$
    – Count Iblis
    4 hours ago










  • $begingroup$
    @CountIblis Didn't catch that one before, thank you!
    $endgroup$
    – Ivan V.
    4 hours ago













2












2








2





$begingroup$


Consider the following definite real integral:
$$I = int_0^infty dx frace^-ix - e^ixx$$



Using the $textSi(x)$ function, I can solve it easily,
$$I = -2i int_0^infty dx frace^-ix - e^ix-2ix = -2i int_0^infty dx fracsinxx = -2i lim_x to infty textSi(x) = -2i left(fracpi2right) = - i pi,$$
simply because I happen to know that $mathrmSi(x)$ asymptotically approaches $pi/2$.



However, if I try to calculate it using the residue theorem, I get the wrong answer, off by a factor of $2$ and I'm not sure if I understand why. Here's the procedure:
$$I= int_0^infty dx frace^-ixx - int_0^infty dx frac e^ixx = colorred-int_-infty^0 dx frace^ixx - int_0^infty dx frac e^ixx
= -int_-infty^infty dx frace^ixx $$

Then I define $$I_epsilon := -int_-infty^infty dx frace^ixx-ivarepsilon$$ for $varepsilon > 0$ so that$$I=lim_varepsilon to 0^+ I_varepsilon.$$
Then I complexify the integration variable and integrate over a D-shaped contour over the upper half of the complex plane. I choose that contour because
$$lim_x to +iinfty frace^ixx-ivarepsilon = 0$$ and it contains the simple pole at $x_0 = i varepsilon$. Using the residue theorem with the contour enclosing $x_0$ $$I_varepsilon = -2 pi i , textRes_x_0 left( frace^ixx-ivarepsilonright) = -2 pi i left( frace^ix1 right)Biggrvert_x=x_0=ivarepsilon=-2 pi i , e^-varepsilon.$$
Therefore,
$$I=lim_varepsilon to 0^+ left( -2 pi i , e^-varepsilon right) = -2pi i.$$



However, that is obviously wrong. Where exactly is the mistake?










share|cite|improve this question









$endgroup$




Consider the following definite real integral:
$$I = int_0^infty dx frace^-ix - e^ixx$$



Using the $textSi(x)$ function, I can solve it easily,
$$I = -2i int_0^infty dx frace^-ix - e^ix-2ix = -2i int_0^infty dx fracsinxx = -2i lim_x to infty textSi(x) = -2i left(fracpi2right) = - i pi,$$
simply because I happen to know that $mathrmSi(x)$ asymptotically approaches $pi/2$.



However, if I try to calculate it using the residue theorem, I get the wrong answer, off by a factor of $2$ and I'm not sure if I understand why. Here's the procedure:
$$I= int_0^infty dx frace^-ixx - int_0^infty dx frac e^ixx = colorred-int_-infty^0 dx frace^ixx - int_0^infty dx frac e^ixx
= -int_-infty^infty dx frace^ixx $$

Then I define $$I_epsilon := -int_-infty^infty dx frace^ixx-ivarepsilon$$ for $varepsilon > 0$ so that$$I=lim_varepsilon to 0^+ I_varepsilon.$$
Then I complexify the integration variable and integrate over a D-shaped contour over the upper half of the complex plane. I choose that contour because
$$lim_x to +iinfty frace^ixx-ivarepsilon = 0$$ and it contains the simple pole at $x_0 = i varepsilon$. Using the residue theorem with the contour enclosing $x_0$ $$I_varepsilon = -2 pi i , textRes_x_0 left( frace^ixx-ivarepsilonright) = -2 pi i left( frace^ix1 right)Biggrvert_x=x_0=ivarepsilon=-2 pi i , e^-varepsilon.$$
Therefore,
$$I=lim_varepsilon to 0^+ left( -2 pi i , e^-varepsilon right) = -2pi i.$$



However, that is obviously wrong. Where exactly is the mistake?







integration residue-calculus






share|cite|improve this question













share|cite|improve this question











share|cite|improve this question




share|cite|improve this question










asked 5 hours ago









Ivan V.Ivan V.

811216




811216







  • 1




    $begingroup$
    math.stackexchange.com/a/2270510/155436
    $endgroup$
    – Count Iblis
    4 hours ago










  • $begingroup$
    @CountIblis Didn't catch that one before, thank you!
    $endgroup$
    – Ivan V.
    4 hours ago












  • 1




    $begingroup$
    math.stackexchange.com/a/2270510/155436
    $endgroup$
    – Count Iblis
    4 hours ago










  • $begingroup$
    @CountIblis Didn't catch that one before, thank you!
    $endgroup$
    – Ivan V.
    4 hours ago







1




1




$begingroup$
math.stackexchange.com/a/2270510/155436
$endgroup$
– Count Iblis
4 hours ago




$begingroup$
math.stackexchange.com/a/2270510/155436
$endgroup$
– Count Iblis
4 hours ago












$begingroup$
@CountIblis Didn't catch that one before, thank you!
$endgroup$
– Ivan V.
4 hours ago




$begingroup$
@CountIblis Didn't catch that one before, thank you!
$endgroup$
– Ivan V.
4 hours ago










3 Answers
3






active

oldest

votes


















3












$begingroup$

You've replaced the converging integral $int_0^infty fracmathrme^-mathrmi x - mathrme^mathrmi xx ,mathrmdx$ with two divergent integrals, $int_0^infty fracmathrme^-mathrmi xx ,mathrmdx$ and $int_0^infty fracmathrme^mathrmi xx ,mathrmdx$. (That something divergent has been introduced is evident in your need to sneak up on a singularity at $0$ that was not in the original integral.)



Also, notice that your D-shaped contour does not go around your freshly minted singularity at $x = 0$. The singularity lands on your contour. See the Sokhotski–Plemelj theorem to find that the multiplier for the residue of the pole is $pm pi mathrmi$, not $pm 2 pi mathrmi$.






share|cite|improve this answer









$endgroup$












  • $begingroup$
    Ah, of course! And thank you for the additional info, very useful.
    $endgroup$
    – Ivan V.
    4 hours ago


















2












$begingroup$

You cannot shift the pole from the integration contour at will. Imagine that you shift it in the lower complex half-plane. Then instead of $-2pi i$ you would obtain for the integral the value $0$!



The correct way to handle the pole is to take the half of its residue value, which is equivalent to bypassing the pole along a tiny semicircle around it (observe that the result does not depend on the choice between upper and lower semicircle).






share|cite|improve this answer











$endgroup$




















    0












    $begingroup$

    There is a problem at the very first step. You cannot split the integral because both integrals are divergent.






    share|cite|improve this answer









    $endgroup$













      Your Answer





      StackExchange.ifUsing("editor", function ()
      return StackExchange.using("mathjaxEditing", function ()
      StackExchange.MarkdownEditor.creationCallbacks.add(function (editor, postfix)
      StackExchange.mathjaxEditing.prepareWmdForMathJax(editor, postfix, [["$", "$"], ["\\(","\\)"]]);
      );
      );
      , "mathjax-editing");

      StackExchange.ready(function()
      var channelOptions =
      tags: "".split(" "),
      id: "69"
      ;
      initTagRenderer("".split(" "), "".split(" "), channelOptions);

      StackExchange.using("externalEditor", function()
      // Have to fire editor after snippets, if snippets enabled
      if (StackExchange.settings.snippets.snippetsEnabled)
      StackExchange.using("snippets", function()
      createEditor();
      );

      else
      createEditor();

      );

      function createEditor()
      StackExchange.prepareEditor(
      heartbeatType: 'answer',
      autoActivateHeartbeat: false,
      convertImagesToLinks: true,
      noModals: true,
      showLowRepImageUploadWarning: true,
      reputationToPostImages: 10,
      bindNavPrevention: true,
      postfix: "",
      imageUploader:
      brandingHtml: "Powered by u003ca class="icon-imgur-white" href="https://imgur.com/"u003eu003c/au003e",
      contentPolicyHtml: "User contributions licensed under u003ca href="https://creativecommons.org/licenses/by-sa/3.0/"u003ecc by-sa 3.0 with attribution requiredu003c/au003e u003ca href="https://stackoverflow.com/legal/content-policy"u003e(content policy)u003c/au003e",
      allowUrls: true
      ,
      noCode: true, onDemand: true,
      discardSelector: ".discard-answer"
      ,immediatelyShowMarkdownHelp:true
      );



      );













      draft saved

      draft discarded


















      StackExchange.ready(
      function ()
      StackExchange.openid.initPostLogin('.new-post-login', 'https%3a%2f%2fmath.stackexchange.com%2fquestions%2f3167734%2freal-integral-using-residue-theorem-why-doesnt-this-work%23new-answer', 'question_page');

      );

      Post as a guest















      Required, but never shown

























      3 Answers
      3






      active

      oldest

      votes








      3 Answers
      3






      active

      oldest

      votes









      active

      oldest

      votes






      active

      oldest

      votes









      3












      $begingroup$

      You've replaced the converging integral $int_0^infty fracmathrme^-mathrmi x - mathrme^mathrmi xx ,mathrmdx$ with two divergent integrals, $int_0^infty fracmathrme^-mathrmi xx ,mathrmdx$ and $int_0^infty fracmathrme^mathrmi xx ,mathrmdx$. (That something divergent has been introduced is evident in your need to sneak up on a singularity at $0$ that was not in the original integral.)



      Also, notice that your D-shaped contour does not go around your freshly minted singularity at $x = 0$. The singularity lands on your contour. See the Sokhotski–Plemelj theorem to find that the multiplier for the residue of the pole is $pm pi mathrmi$, not $pm 2 pi mathrmi$.






      share|cite|improve this answer









      $endgroup$












      • $begingroup$
        Ah, of course! And thank you for the additional info, very useful.
        $endgroup$
        – Ivan V.
        4 hours ago















      3












      $begingroup$

      You've replaced the converging integral $int_0^infty fracmathrme^-mathrmi x - mathrme^mathrmi xx ,mathrmdx$ with two divergent integrals, $int_0^infty fracmathrme^-mathrmi xx ,mathrmdx$ and $int_0^infty fracmathrme^mathrmi xx ,mathrmdx$. (That something divergent has been introduced is evident in your need to sneak up on a singularity at $0$ that was not in the original integral.)



      Also, notice that your D-shaped contour does not go around your freshly minted singularity at $x = 0$. The singularity lands on your contour. See the Sokhotski–Plemelj theorem to find that the multiplier for the residue of the pole is $pm pi mathrmi$, not $pm 2 pi mathrmi$.






      share|cite|improve this answer









      $endgroup$












      • $begingroup$
        Ah, of course! And thank you for the additional info, very useful.
        $endgroup$
        – Ivan V.
        4 hours ago













      3












      3








      3





      $begingroup$

      You've replaced the converging integral $int_0^infty fracmathrme^-mathrmi x - mathrme^mathrmi xx ,mathrmdx$ with two divergent integrals, $int_0^infty fracmathrme^-mathrmi xx ,mathrmdx$ and $int_0^infty fracmathrme^mathrmi xx ,mathrmdx$. (That something divergent has been introduced is evident in your need to sneak up on a singularity at $0$ that was not in the original integral.)



      Also, notice that your D-shaped contour does not go around your freshly minted singularity at $x = 0$. The singularity lands on your contour. See the Sokhotski–Plemelj theorem to find that the multiplier for the residue of the pole is $pm pi mathrmi$, not $pm 2 pi mathrmi$.






      share|cite|improve this answer









      $endgroup$



      You've replaced the converging integral $int_0^infty fracmathrme^-mathrmi x - mathrme^mathrmi xx ,mathrmdx$ with two divergent integrals, $int_0^infty fracmathrme^-mathrmi xx ,mathrmdx$ and $int_0^infty fracmathrme^mathrmi xx ,mathrmdx$. (That something divergent has been introduced is evident in your need to sneak up on a singularity at $0$ that was not in the original integral.)



      Also, notice that your D-shaped contour does not go around your freshly minted singularity at $x = 0$. The singularity lands on your contour. See the Sokhotski–Plemelj theorem to find that the multiplier for the residue of the pole is $pm pi mathrmi$, not $pm 2 pi mathrmi$.







      share|cite|improve this answer












      share|cite|improve this answer



      share|cite|improve this answer










      answered 4 hours ago









      Eric TowersEric Towers

      33.3k22370




      33.3k22370











      • $begingroup$
        Ah, of course! And thank you for the additional info, very useful.
        $endgroup$
        – Ivan V.
        4 hours ago
















      • $begingroup$
        Ah, of course! And thank you for the additional info, very useful.
        $endgroup$
        – Ivan V.
        4 hours ago















      $begingroup$
      Ah, of course! And thank you for the additional info, very useful.
      $endgroup$
      – Ivan V.
      4 hours ago




      $begingroup$
      Ah, of course! And thank you for the additional info, very useful.
      $endgroup$
      – Ivan V.
      4 hours ago











      2












      $begingroup$

      You cannot shift the pole from the integration contour at will. Imagine that you shift it in the lower complex half-plane. Then instead of $-2pi i$ you would obtain for the integral the value $0$!



      The correct way to handle the pole is to take the half of its residue value, which is equivalent to bypassing the pole along a tiny semicircle around it (observe that the result does not depend on the choice between upper and lower semicircle).






      share|cite|improve this answer











      $endgroup$

















        2












        $begingroup$

        You cannot shift the pole from the integration contour at will. Imagine that you shift it in the lower complex half-plane. Then instead of $-2pi i$ you would obtain for the integral the value $0$!



        The correct way to handle the pole is to take the half of its residue value, which is equivalent to bypassing the pole along a tiny semicircle around it (observe that the result does not depend on the choice between upper and lower semicircle).






        share|cite|improve this answer











        $endgroup$















          2












          2








          2





          $begingroup$

          You cannot shift the pole from the integration contour at will. Imagine that you shift it in the lower complex half-plane. Then instead of $-2pi i$ you would obtain for the integral the value $0$!



          The correct way to handle the pole is to take the half of its residue value, which is equivalent to bypassing the pole along a tiny semicircle around it (observe that the result does not depend on the choice between upper and lower semicircle).






          share|cite|improve this answer











          $endgroup$



          You cannot shift the pole from the integration contour at will. Imagine that you shift it in the lower complex half-plane. Then instead of $-2pi i$ you would obtain for the integral the value $0$!



          The correct way to handle the pole is to take the half of its residue value, which is equivalent to bypassing the pole along a tiny semicircle around it (observe that the result does not depend on the choice between upper and lower semicircle).







          share|cite|improve this answer














          share|cite|improve this answer



          share|cite|improve this answer








          edited 4 hours ago

























          answered 4 hours ago









          useruser

          6,09811031




          6,09811031





















              0












              $begingroup$

              There is a problem at the very first step. You cannot split the integral because both integrals are divergent.






              share|cite|improve this answer









              $endgroup$

















                0












                $begingroup$

                There is a problem at the very first step. You cannot split the integral because both integrals are divergent.






                share|cite|improve this answer









                $endgroup$















                  0












                  0








                  0





                  $begingroup$

                  There is a problem at the very first step. You cannot split the integral because both integrals are divergent.






                  share|cite|improve this answer









                  $endgroup$



                  There is a problem at the very first step. You cannot split the integral because both integrals are divergent.







                  share|cite|improve this answer












                  share|cite|improve this answer



                  share|cite|improve this answer










                  answered 4 hours ago









                  Kavi Rama MurthyKavi Rama Murthy

                  71k53170




                  71k53170



























                      draft saved

                      draft discarded
















































                      Thanks for contributing an answer to Mathematics Stack Exchange!


                      • Please be sure to answer the question. Provide details and share your research!

                      But avoid


                      • Asking for help, clarification, or responding to other answers.

                      • Making statements based on opinion; back them up with references or personal experience.

                      Use MathJax to format equations. MathJax reference.


                      To learn more, see our tips on writing great answers.




                      draft saved


                      draft discarded














                      StackExchange.ready(
                      function ()
                      StackExchange.openid.initPostLogin('.new-post-login', 'https%3a%2f%2fmath.stackexchange.com%2fquestions%2f3167734%2freal-integral-using-residue-theorem-why-doesnt-this-work%23new-answer', 'question_page');

                      );

                      Post as a guest















                      Required, but never shown





















































                      Required, but never shown














                      Required, but never shown












                      Required, but never shown







                      Required, but never shown

































                      Required, but never shown














                      Required, but never shown












                      Required, but never shown







                      Required, but never shown







                      Popular posts from this blog

                      Magento 2 - Add success message with knockout Planned maintenance scheduled April 23, 2019 at 23:30 UTC (7:30pm US/Eastern) Announcing the arrival of Valued Associate #679: Cesar Manara Unicorn Meta Zoo #1: Why another podcast?Success / Error message on ajax request$.widget is not a function when loading a homepage after add custom jQuery on custom themeHow can bind jQuery to current document in Magento 2 When template load by ajaxRedirect page using plugin in Magento 2Magento 2 - Update quantity and totals of cart page without page reload?Magento 2: Quote data not loaded on knockout checkoutMagento 2 : I need to change add to cart success message after adding product into cart through pluginMagento 2.2.5 How to add additional products to cart from new checkout step?Magento 2 Add error/success message with knockoutCan't validate Post Code on checkout page

                      Fil:Tokke komm.svg

                      Where did Arya get these scars? Unicorn Meta Zoo #1: Why another podcast? Announcing the arrival of Valued Associate #679: Cesar Manara Favourite questions and answers from the 1st quarter of 2019Why did Arya refuse to end it?Has the pronunciation of Arya Stark's name changed?Has Arya forgiven people?Why did Arya Stark lose her vision?Why can Arya still use the faces?Has the Narrow Sea become narrower?Does Arya Stark know how to make poisons outside of the House of Black and White?Why did Nymeria leave Arya?Why did Arya not kill the Lannister soldiers she encountered in the Riverlands?What is the current canonical age of Sansa, Bran and Arya Stark?